LSAT and Law School Admissions Forum

Get expert LSAT preparation and law school admissions advice from PowerScore Test Preparation.

 Basia W
  • Posts: 108
  • Joined: Jun 19, 2014
|
#15158
Yes it does thank you! I forgot that the contrapositive could form part of the first section of the diagram,

thank you,

best,

Basia
 prep88
  • Posts: 37
  • Joined: Jan 20, 2015
|
#18480
Can anyone, please, explain the conditional diagram here? My diagram was "Aleviate injustice------> Redistribute wealth"; quite different from homework explanation.

And also, why is this a Must Be True, not a Main Point question?

-Thanks
 Andrew Ash
PowerScore Staff
  • PowerScore Staff
  • Posts: 32
  • Joined: Sep 15, 2014
|
#18483
Hi Prep,

Good to hear from you!

I agree that the way we explain this problem in the homework is a little confusing. Your diagram of the "unless" statement is spot on: Able to alleviate injustice :arrow: redistribute wealth. The two other conditional statements in the stimulus are as they appear in the online explanation: not able to alleviate injustice :arrow: intolerable inequities, and intolerable inequities :arrow: violence.

The key step here, and our explanation glosses over it a bit, is to take the contrapositive of the first "unless" statement, which then becomes not redistribute :arrow: not able to alleviate injustice. Once you do that, the statements all chain together nicely to suggest that we need to redistribute wealth in order to avoid violence.

As for why this is a Must Be True question, and not a Main Point, see my reply to your other post.

Let me know if you have any questions!

Thanks,
Andrew
 a.lsat
  • Posts: 9
  • Joined: Aug 05, 2015
|
#19466
I was a bit stuck with the conditional reasoning in the stimulus even though I got the answer right. Can you kindly help me with that?

This is how I approach the stimulus:

(Premises)
Alleviate economic injustice
      and         :arrow: Nation redistribute wealth
~Intolerable economic injustice

Intolerable economic injustice :arrow: Violence

(Premises- combined)
         Alleviate economic injustice
              and         :arrow: Nation redistribute wealth
~Violence :arrow: ~Intolerable economic injustice

(Conclusion)
~Violence 

--

Are my inferences above correct? If they are correct, then I was a bit stuck with the correct answer, since one of the sufficient condition is not fulfilled.

Let's say that
A :arrow: B

B
and :arrow: D
C

If we know that A exists, we can infer that B also exists, yet not sure if C exists. Can we thus inter that D must also exist? Or, is there anything wring with my approach in the conditional reasoning?

Thanks!
 David Boyle
PowerScore Staff
  • PowerScore Staff
  • Posts: 836
  • Joined: Jun 07, 2013
|
#19496
a.lsat wrote:I was a bit stuck with the conditional reasoning in the stimulus even though I got the answer right. Can you kindly help me with that?

This is how I approach the stimulus:

(Premises)
Alleviate economic injustice
      and         :arrow: Nation redistribute wealth
~Intolerable economic injustice

Intolerable economic injustice :arrow: Violence

(Premises- combined)
         Alleviate economic injustice
              and         :arrow: Nation redistribute wealth
~Violence :arrow: ~Intolerable economic injustice

(Conclusion)
~Violence 

--

Are my inferences above correct? If they are correct, then I was a bit stuck with the correct answer, since one of the sufficient condition is not fulfilled.

Let's say that
A :arrow: B

B
and :arrow: D
C

If we know that A exists, we can infer that B also exists, yet not sure if C exists. Can we thus inter that D must also exist? Or, is there anything wring with my approach in the conditional reasoning?

Thanks!
Hello a.lsat,

No, "Can we thus inter that D must also exist?" seems wrong. If you don't have C, you can't assume you have D.
I'm not sure how you are connecting that idea to what is above. The economic etc. things you connect at the top, out of the stimulus, all seem to connect fairly well, so I'm not sure what other point you're trying to make. If you could explain further, that would be helpful.
--One point of clarification: "Alleviate economic injustice" may be sort of synonymous with "~Intolerable economic injustice", instead of being a truly separate condition, if you see what I mean. Thanks,

Hope this helps,
David
 Mustafaabdulmalek
  • Posts: 19
  • Joined: Nov 17, 2015
|
#21221
answer choice E isn't the redistribution of wealth a necessary condition ??
because of Required ??
 Clay Cooper
PowerScore Staff
  • PowerScore Staff
  • Posts: 241
  • Joined: Jul 03, 2015
|
#21234
Hi Mustafa,

Thanks for your question. The short answer is that, no, answer choice E is an example of mistaken reversal and is thus incorrect. A more in depth explanation follows below. But be encouraged, this stimulus presents a real challenge to our ability to understand and correctly diagram conditional reasoning.

Let’s first break down the stimulus and see if we can understand each piece individually – maybe then we can connect them.

The first sentence, “Unless…inequities.” can be diagrammed using the unless formula: we first take the condition modified by the unless (in this case, our nation redistributes wealth, or RW) and make it the necessary condition (or, in graphical terms, put it on the right side of the arrow). Then we take the other condition in the sentence (our system will lead to intolerable economic inequities), negate it and make it the sufficient (or left-hand) term in our rule. What we end up with looks like this: ~IEI --> RW and reads, “If intolerable economic inequities do not occur, then we redistributed wealth.”

The next sentence presents another conditional rule, though it uses one of the terms we have already seen. Since this sentence is expressed as an “if…(then)” statement, it is simple to diagram and understand: the condition modified by the “if” is on the left and is the sufficient condition, while the other condition is the right-hand (necessary) condition. What we end up with looks like this: IEI --> VSR and reads, “if economic inequities are intolerable, violent social reform will occur.”

The third and final piece states that any condition which would set off this chain of events (to wit, by causing intolerable economic inequities, or IEI) must be avoided. Since we know from the contrapositive of our first piece that ~RW --> IEI, we must avoid ~RW, or, in simpler terms, society must redistribute wealth. This is why answer choice B is correct.

Answer choice E, however, mistakenly negates our first piece of evidence (the one that looked like this: ~IEI --> RW). First, let’s diagram E and see if we can make sense of it in the terms we have already been using. To begin, “conditions of economic justice” sounds like ~IEI, or like the absence of intolerable economic inequities; the other term is more obviously the same as we have seen: redistributing wealth, or RW. However, the sentence in E, when diagrammed, looks like this: RW --> ~IEI. This diagram reflects the answer choice because, according to this answer choice, since RW is all that is required to create ~IEI, then RW happening is sufficient to prove that ~IEI follows, hence RW --> ~IEI. However, when we compare this diagram with our diagram of the first sentence of the stimulus, we see that the former is a mistaken reversal of the latter and therefore is incorrect.

Thanks for your question, keep working hard!
 Mustafaabdulmalek
  • Posts: 19
  • Joined: Nov 17, 2015
|
#21241
Thank you for your answer
Answer choice B make more sense but what I can not understand here is
All that is required to create conditions of economic justice is the redistribution of wealth = all that is required is the redistribution of wealth to creat condition of economic justice

And the diagram will be Justice ---> redistribution of wealth
And the redistribution of wealth is necessary
In my opinion is the answer B is right and answer E also right but since the question about the conclusion it does make more sense that B is better but not that E a mistaken
Regards,
 Mustafaabdulmalek
  • Posts: 19
  • Joined: Nov 17, 2015
|
#21242
And. Isn't "required" indicate a necessary condition ?
 Steve Stein
PowerScore Staff
  • PowerScore Staff
  • Posts: 1153
  • Joined: Apr 11, 2011
|
#21249
Hey Mustafa,

That's a good question. You're right about the fact that the word "required" is usually associated with a necessary condition—indeed, the word is actually synonymous with “necessary.”

That answer choice, however, says that wealth distribution is “all that is required.” If you have all that is required to do something, then what you have is sufficient. Consider the following example:
  • All you need to enter the theater is a ticket. A ticket is all you need for entry.
Another way to phrase this would be “a ticket is sufficient for entry.” Ticket :arrow: Entry

So, if "all that is required" to create justice is wealth distribution, this means that wealth distribution is sufficient to create justice:
  • Wealth redistribution :arrow: Create Justice
The moral of the story: If something is required, it is necessary. If something is "all that's required," that's sufficient.

Tricky! I hope this is helpful—please let me know whether this is clear—thanks!

~Steve

Get the most out of your LSAT Prep Plus subscription.

Analyze and track your performance with our Testing and Analytics Package.